Đến nội dung

Hình ảnh

$\left\lfloor\text{Phần nguyên}\right\rfloor$

Floor Function Phần nguyên

  • Please log in to reply
Chủ đề này có 26 trả lời

#1
hxthanh

hxthanh

    Tín đồ $\sum$

  • Hiệp sỹ
  • 3921 Bài viết
Định nghĩa:
Phần nguyên của số thực x, kí hiệu là $\lfloor x\rfloor$ là: "số nguyên lớn nhất nhỏ hơn hoặc bằng x"
VD: $\lfloor 3.2\rfloor=3;\;\lfloor 3\rfloor=3;\;\lfloor -1.5\rfloor=-2;\; \lfloor\sqrt{3}\rfloor= 1$; v.v...
Các phát biểu sau là tương đương:
$ \left\{\begin{array}{l}n \in \mathbb{Z}\\ n<x\end{array}\right.\Rightarrow n \leq \lfloor x\rfloor.$

$\lfloor x\rfloor \leq x < \lfloor x\rfloor+1$ :Leftrightarrow $x-1 < \lfloor x\rfloor \leq x$

Ta cũng gọi $\{x\} =x-\lfloor x\rfloor$ là phần lẻ của số thực x
Từ định nghĩa phần nguyên ta có
$0 \leq \{x\} <1$
Một số kết quả cơ bản:
  • $\lfloor x+n\rfloor=\lfloor x\rfloor+n$, với $n \in \mathbb{Z}$
  • $\lfloor -n\rfloor=-\lfloor n\rfloor$ ,với $n \in \mathbb{Z}$
  • $\lfloor -x\rfloor=-1-\lfloor x\rfloor$, với $x>0$ và $x \not\in \mathbb{Z}$
  • $x>y \Rightarrow \lfloor x\rfloor \geq \lfloor y\rfloor$
  • $\lfloor x \rfloor +\lfloor y\rfloor \leq\lfloor x+y\rfloor \leq\lfloor x \rfloor +\lfloor y\rfloor+1$
Với phép chia giữa các số nguyên dương:
$n=p.m+q$ với $0 \leq q \leq m-1$ (n chia m được p dư q)
Ta có: thương là $p=\left\lfloor\dfrac{n}{m}\right\rfloor$
phần dư (mod m) là $q=n-m\left\lfloor\dfrac{n}{m}\right\rfloor$
Hay đơn giản là: $0 \leq n-m\left\lfloor\dfrac{n}{m}\right\rfloor \leq m-1$
(Công thức này rất có ích trong việc biểu diễn chung 1 công thức tổng quát cho m trường hợp cần xét)

Hệ quả: $\left\lfloor\dfrac{\left\lfloor\dfrac{x}{n}\right\rfloor}{m}\right\rfloor=\left\lfloor\dfrac{x}{m.n}\right\rfloor$ Với mọi số nguyên dương $m, n$
(Các bạn tự CM nhé!)
Nghỉ cái đã: Bài sau mình sẽ cho các bạn một số bài tập VD.

#2
hxthanh

hxthanh

    Tín đồ $\sum$

  • Hiệp sỹ
  • 3921 Bài viết
Chúng ta sẽ cùng nhau tìm hiểu bài toán sau đây nhé!
Bài toán:
Tính
$S_n=\sum\limits_{k=0}^{n} \left\lfloor{\dfrac{k^2-k}{3}}\right\rfloor$
Lời giải:
Ta thấy các số hạng của $S_n$ là phép chia cho 3 lấy nguyên. Vì thế ta đặt
$n=3p+q$ với $0 \leq q \leq 2$
Ta có:
$\begin{align*}S_{3p+2}\ &= S_{3p+1}+\left\lfloor{\dfrac{(3p+2)^2-(3p+2)}{3}}\right\rfloor\\ &= S_{3p+1}+\left\lfloor{\dfrac{9p^2+12p+4-3p-2}{3}}\right\rfloor\\ &= S_{3p+1}+ 3p^2+3p\end{align*}$
$\begin{align*}S_{3p+1}\ &= S_{3p}+\left\lfloor{\dfrac{(3p+1)^2-(3p+1)}{3}}\right\rfloor\\ &= S_{3p}+\left\lfloor{\dfrac{9p^2+6p+1-3p-1}{3}}\right\rfloor\\ &= S_{3p}+ 3p^2+p\end{align*}$
$\begin{align*}S_{3p}\ &= S_{3p-1}+\left\lfloor{\dfrac{(3p)^2-3p}{3}}\right\rfloor\\ &= S_{3(p-1)+2}+3p^2-p\end{align*}$
Từ 3 đẳng thức trên suy ra:
$S_{3p+2}=S_{3(p-1)+2}+(3p^2+3p)+(3p^2+p)+3p^2-p$

$\begin{align*}S_{3p+2} &= S_{3(p-1)+2}+9p^2+3p\\S_{3(p-1)+2} &= S_{3(p-2)+2}+9(p-1)^2+3(p-1)\\ ... \\ S_5 &=S_2+9.1^2+3.1 \end{align*}$
Cộng các đẳng thức theo từng vế, suy ra
$S_{3p+2}=9\sum\limits_{i=1}^{p} i^2+3\sum\limits_{i=1}^{p} i$ (do $S_2=0$)
:vdots (sau khi tính và rút gọn)
$S_{3p+2}=3p^3+6p^2+3p$ (1)
$S_{3p+1}=S_{3p+2}-(3p^2+3p)=3p^3+3p^2$ (2)
$S_{3p}=S_{3p+1}-(3p^2+p)=3p^3-p$ (3)

Bây giờ ta sẽ dùng $\left\lfloor{...}\right\rfloor$ để gộp (1), (2)(3) thành 1 công thức!
p thì rõ rồi $p=\left\lfloor{\dfrac{n}{3}}\right\rfloor$

[NHÁP]
Hệ số $p^2$ theo thứ tự trong (3) - (1) là {0,3,6}
Hệ số $p$ theo thứ tự trong (3) - (1) là {-1,0,3}
Theo thứ tự $n=3p$ đến $n=3p+2$ thì $q =$ {0,1,2}
:vdots {0,3,6}$=3q$ và {-1,0,3}$=q^2-1$
Với $q=n-3\left\lfloor{\dfrac{n}{3}}\right\rfloor$
[\NHÁP XONG]

Công thức ta cần tìm là

$S_n=3\left\lfloor{\dfrac{n}{3}}\right\rfloor^3+3(n-3\left\lfloor{\dfrac{n}{3}}\right\rfloor)\left\lfloor{\dfrac{n}{3}}\right\rfloor^2+((n-3\left\lfloor{\dfrac{n}{3}}\right\rfloor)^2-1)\left\lfloor{\dfrac{n}{3}}\right\rfloor$
nhân ra rồi rút gọn lại
$S_n=3\left\lfloor\dfrac{n}{3}\right\rfloor^3-3n\left\lfloor\dfrac{n}{3}\right\rfloor^2+(n^2-1)\left\lfloor\dfrac{n}{3}\right\rfloor$

#3
hxthanh

hxthanh

    Tín đồ $\sum$

  • Hiệp sỹ
  • 3921 Bài viết
He he... Mình search trên Wikipedia tìm được 2 bài khá hay
CMR:
  • $\lfloor\dfrac{n}{3}\rfloor+\lfloor\dfrac{n+2}{6}\rfloor+\lfloor\dfrac{n+4}{6}\rfloor=\lfloor\dfrac{n}{2}\rfloor+\lfloor\dfrac{n+3}{6}\rfloor$ với mọi $n\in\mathbb{Z}$
  • $\lfloor \sqrt{n}+\sqrt{n+1}\rfloor=\lfloor \sqrt{4n+2}\rfloor$ với mọi $n \in \mathbb{N}$


#4
Ho pham thieu

Ho pham thieu

    Lính mới

  • Thành viên
  • 440 Bài viết

He he... Mình search trên Wikipedia tìm được 2 bài khá hay
CMR:
[*]$\lfloor \sqrt{n}+\sqrt{n+1}\rfloor=\lfloor \sqrt{4n+2}\rfloor$ với mọi $n \in \math\bb{N}$


$( \sqrt{n}+\sqrt{n+1})^2=2n+1+2\sqrt{n(n+1)}<4n+2 $
$\Rightarrow \sqrt{n}+\sqrt{n+1} < \sqrt{4n+2}$

G/s :) m :sum N sao cho $\sqrt{n}+\sqrt{n+1} <m \leq \sqrt{4n+2} \Leftrightarrow 2n+1+2\sqrt{n(n+1)} <m \leq 4n+2$

$ \Rightarrow 2n<2\sqrt{n(n+1)}<m^2-(2n+1) \leq 2n+1 \Rightarrow m^2=4n+2$ Vô lí, vì scp chia cho 4 có số dư 0 hoặc 1

Không tồn tại số nguyên nằm trong nửa khoảng của 2 số đó.

Vậy $\Rightarrow \lfloor \sqrt{n}+\sqrt{n+1}\rfloor = \lfloor \sqrt{4n+2}\rfloor$

Đã sửa

Bài viết đã được chỉnh sửa nội dung bởi Ho pham thieu: 17-12-2010 - 13:00

Nếu thấy bài viết nào hay thì cách tốt nhất để cám ơn là hãy click vào "nút" thanks cho người đó.
I love football musics.

#5
hxthanh

hxthanh

    Tín đồ $\sum$

  • Hiệp sỹ
  • 3921 Bài viết

$ \Rightarrow 2n<2\sqrt{n(n+1)}<m^2-(2n+1)<2n+1 \Rightarrow m^2=4n+2$ Vô lí, vì scp chia cho 4 có số dư 0 hoặc 1

Chỗ đó có vấn đề
$ \Rightarrow 2n<2\sqrt{n(n+1)}<m^2-(2n+1)<2n+1 \Rightarrow m^2-(2n+1)\not\in\mathbb{Z}$
(Vì 2n,2n+1 là 2 số nguyên liên tiếp nên ở giữa nó không có số nguyên nào cả!)
vô lý! $\Rightarrow$ ĐPCM
----------
Ai chém nốt được bài 1 nào ???

#6
hxthanh

hxthanh

    Tín đồ $\sum$

  • Hiệp sỹ
  • 3921 Bài viết
Thôi để mình "chém" luôn vậy!

CMR:

  • $\left\lfloor\dfrac{n}{3}\right\rfloor+\left\lfloor\dfrac{n+2}{6}\right\rfloor+\left\lfloor\dfrac{n+4}{6}\right\rfloor=\left\lfloor\dfrac{n}{2}\right\rfloor+\left\lfloor\dfrac{n+3}{6}\right\rfloor$ với mọi $n\in\mathbb{Z}$

Đặt $n=6p+r $
trong đó
$p=\left\lfloor\dfrac{n}{6}\right\rfloor $
$r=n-6\left\lfloor\dfrac{n}{6}\right\rfloor=\{0,1,2,3,4,5\} $ (tương ứng với 6 trường hợp)
Ta có
$ \left\lfloor\dfrac{n}{3}\right\rfloor=\left\lfloor 2p+\dfrac{r}{3}\right\rfloor=2p+\left\lfloor\dfrac{r}{3}\right\rfloor=2p+\{0,0,0,1,1,1\}$

$ \left\lfloor\dfrac{n+2}{6}\right\rfloor=\left\lfloor p+\dfrac{r+2}{6}\right\rfloor=p+\left\lfloor\dfrac{r+2}{6}\right\rfloor=p+\{0,0,0,0,1,1\}$

$ \left\lfloor\dfrac{n+4}{6}\right\rfloor=\left\lfloor p+\dfrac{r+4}{6}\right\rfloor=p+\left\lfloor\dfrac{r+4}{6}\right\rfloor=p+\{0,0,1,1,1,1\}$
Suy ra
$VT=2p+p+p+\{0,0,0,1,1,1\}+\{0,0,0,0,1,1\}+\{0,0,1,1,1,1\}=4p+\{0,0,1,2,3,3\}$

Tương tự
$ \left\lfloor\dfrac{n}{2}\right\rfloor=\left\lfloor 3p+\dfrac{r}{2}\right\rfloor=3p+\left\lfloor\dfrac{r}{2}\right\rfloor=3p+\{0,0,1,1,2,2\}$

$ \left\lfloor\dfrac{n+3}{6}\right\rfloor=\left\lfloor p+\dfrac{r+3}{6}\right\rfloor=p+\left\lfloor\dfrac{r+3}{6}\right\rfloor=p+\{0,0,0,1,1,1\}$
Suy ra
$VP=3p+p+\{0,0,1,1,2,2\}+\{0,0,0,1,1,1\}=4p+\{0,0,1,2,3,3\}$

ĐPCM

#7
Ho pham thieu

Ho pham thieu

    Lính mới

  • Thành viên
  • 440 Bài viết

Chỗ đó có vấn đề
$ \Rightarrow 2n<2\sqrt{n(n+1)}<m^2-(2n+1)<2n+1 \Rightarrow m^2-(2n+1)\not\in\math\bb{Z}$
(Vì 2n,2n+1 là 2 số nguyên liên tiếp nên ở giữa nó không có số nguyên nào cả!)
:D vô lý :D ĐPCM
----------
Ai chém nốt được bài 1 nào ???


Sửa rồi, e gõ nhầm. ok
Nếu thấy bài viết nào hay thì cách tốt nhất để cám ơn là hãy click vào "nút" thanks cho người đó.
I love football musics.

#8
hxthanh

hxthanh

    Tín đồ $\sum$

  • Hiệp sỹ
  • 3921 Bài viết

:lol:

ĐỊNH LÝ HERMITE
Với $\,n\,$ nguyên dương, $\,x\,$ là số thực bất kỳ ta có:
$\color{blue}{\left\lfloor nx \right\rfloor=\left\lfloor x \right\rfloor+\left\lfloor x+\dfrac{1}{n} \right\rfloor+...+\left\lfloor x+\dfrac{n-1}{n} \right\rfloor}$

#9
hxthanh

hxthanh

    Tín đồ $\sum$

  • Hiệp sỹ
  • 3921 Bài viết
:lol:+-->
Trích dẫn(:lol:)
ĐỊNH LÝ HERMITE
Với $\,n\,$ nguyên dương, $\,x\,$ là số thực bất kỳ ta có:
$\color{blue}{\lfloor nx \rfloor=\lfloor x \rfloor+\lfloor x+\dfrac{1}{n} \rfloor+...+\lfloor x+\dfrac{n-1}{n} \rfloor}$[/quote]
Định lý này có cách chứng minh khá độc đáo
Xét hàm $f(x)=\lfloor x \rfloor+\lfloor x+\dfrac{1}{n} \rfloor+...+\lfloor x+\dfrac{n-1}{n} \rfloor\:-\:\lfloor nx \rfloor$
Ta có: $f(x+\dfrac{1}{n})=\lfloor x+\dfrac{1}{n} \rfloor+...+\lfloor x+\dfrac{n-1}{n}\rfloor+\lfloor x+1\rfloor \:-\:\lfloor nx+1 \rfloor$
:lol: $f(x+\dfrac{1}{n})=f(x)$
Vậy $f(x)\:$ là hàm tuần hoàn với chu kỳ $\dfrac{1}{n}\:$
Ta có $f(\dfrac{1}{n})=f(0)=0$
Xét trên một chu kỳ $0 \leq x < \dfrac{1}{n}$ thì các giá trị $\lfloor x\rfloor;\;\lfloor x+\dfrac{1}{n}\rfloor;\;...\lfloor x+\dfrac{n-1}{n}\rfloor;\;\lfloor nx\rfloor;\;$ đều bằng $0$

Vậy $\:f(x)=0,\;\;\forall x$

Định lý được chứng minh

(Còn cách chứng minh nào đơn giản hơn không nhỉ?)

Bài viết đã được chỉnh sửa nội dung bởi hxthanh: 03-01-2011 - 11:36


#10
hxthanh

hxthanh

    Tín đồ $\sum$

  • Hiệp sỹ
  • 3921 Bài viết

Bài toán khác khó hơn cái này tí!
Chứng minh với $ x $ nguyên dương thì
$ \left\lfloor \sqrt{x}+\sqrt{x+1}+\sqrt{x+2}\right\rfloor = \left\lfloor \sqrt{9x+8}\right\rfloor $

Ta sẽ chứng minh rằng $\forall x \geq 1$
$\sqrt{9x+8} < \sqrt{x}+\sqrt{x+1}+\sqrt{x+2} < \sqrt{9x+9}$
Sau đó suy ra
$\left\lfloor\sqrt{x}+\sqrt{x+1}+\sqrt{x+2}\right\rfloor=\left\lfloor\sqrt{9x+8}\right\rfloor$

Thật vậy:
Theo Cauchy - Schwarz
$1.\sqrt{x}+1.\sqrt{x+1}+1.\sqrt{x+2} < \sqrt{(1^2+1^2+1^2)(x+x+1+x+2)}=\sqrt{9x+9}$

Ta cần chứng minh
$\;\;\;\sqrt{x}+\sqrt{x+1}+\sqrt{x+2} > \sqrt{9x+8}$ Bình phương 2 vế
$\Leftrightarrow VT=3x+3+2(\sqrt{x (x+1)}+\sqrt{x (x+2)}+\sqrt{(x+1)(x+2)}) > 9x+8$
Có thể kiểm tra với x=1, x=2 thì BĐT trên đúng
Với $x \geq 3$; Theo BĐT AM-GM thì
$2\sqrt{x (x+1)}\ge \dfrac{4x (x+1)}{2x+1}=2x+\dfrac{2x}{2x+1}$
$2\sqrt{x (x+2)}\ge \dfrac{4x (x+2)}{2x+2}=2x+\dfrac{2x}{x+1}$
$2\sqrt{(x+1)(x+2)}\ge \dfrac{4(x+1)(x+2)}{2x+3}=2x+2+\dfrac{2x+2}{2x+3}$
$\Rightarrow VT\ge 9x+5+\dfrac{2x}{2x+1}+\dfrac{2x}{x+1}+\dfrac{2x+2}{2x+3}$
Vậy chỉ cần CM
$\dfrac{2x}{2x+1}+\dfrac{2x}{x+1}+\dfrac{2x+2}{2x+3}>3\;\;\forall x \ge 3$
Biến đổi tương đương đưa về việc chứng minh BĐT $f(x)=4x^3-13x-7>0\;\;\forall x \ge 3$
Việc này không khó!

#11
hxthanh

hxthanh

    Tín đồ $\sum$

  • Hiệp sỹ
  • 3921 Bài viết
:delta+-->
Trích dẫn(:delta)
ĐỊNH LÝ HERMITE
Với $\,n\,$ nguyên dương, $\,x\,$ là số thực bất kỳ ta có:
$\mathbf{\color{blue}{\lfloor nx \rfloor=\lfloor x \rfloor+\lfloor x+\dfrac{1}{n} \rfloor+...+\lfloor x+\dfrac{n-1}{n} \rfloor}}$[/quote]
Một số bài tập áp dụng:
Với $\:i,\:j,\:k,\:n\:\in \mathbb{N}^{*},\;\;x\in \mathbb{R},\:$ hãy tính các tổng sau

$\;\;a.\;\;\sum_{0\le i<j\le n}\lfloor \dfrac{x+i}{j}\rfloor$

$\;\;b.\;\;\lfloor \dfrac{n+1}{2}\rfloor+\lfloor \dfrac{n+2}{2^2}\rfloor+...+\lfloor \dfrac{n+2^k}{2^{k+1}}\rfloor+...$

$\;\;c.\;\;\sum_{k=0}^{2009}(\lfloor \dfrac{3^k+2010}{3^{k+1}}\rfloor+\lfloor \dfrac{2010-3^k}{3^{k+1}}\rfloor)$

#12
hiep ga

hiep ga

    Sĩ quan

  • Thành viên
  • 428 Bài viết

Bài toán khác khó hơn cái này tí!
Chứng minh với $ x $ nguyên dương thì
$ [ \sqrt{x}+\sqrt{x+1}+\sqrt{x+2} ] = [\sqrt{9x+8}] $

Bài này hình như có trong VL và TT số mới nhất or số gần mới nhất ấy :delta

Poof


#13
hxthanh

hxthanh

    Tín đồ $\sum$

  • Hiệp sỹ
  • 3921 Bài viết
Bạn tran nguyen quoc cuong giải hơi tham quá. :delta
Trình bày hơi vắn tắt nhìn rối quá!

Bài a. Đúng rồi. ĐS $n\lfloor x\rfloor$
Bài b. Kết quả là $n$ (số hạng còn lại tiến tới 0 khi k đủ lớn)
Bài c. Kết quả đúng là $0$ nhưng làm hơi ẩu (k chạy từ 0 đến 2009 không phải 2010)

Dưới đây là lời giải của mình:
Câu a
$\sum_{0\le i<j\le n }\lfloor \dfrac{x+i}{j}\rfloor=\sum_{0\le i<j\le n-1 }\lfloor \dfrac{x+i}{j}\rfloor+\sum_{i=0}^{n-1} \lfloor \dfrac{x+i}{n}\rfloor\\=\sum_{0\le i<j\le n-1 }\lfloor \dfrac{x+i}{j}\rfloor+\lfloor n.\dfrac{x}{n}\rfloor\\=\sum_{0\le i<j\le n-1 }\lfloor \dfrac{x+i}{j}\rfloor+\lfloor x \rfloor\; \Rightarrow U_n=U_{n-1}+\lfloor x \rfloor $ (theo định lý Hermite)
Với $ U_n=\sum_{0\le i<j\le n }\lfloor \dfrac{x+i}{j}\rfloor $ và $ U_1=\lfloor x \rfloor $ $ \Rightarrow U_n=n\lfloor x \rfloor $

Câu b
Áp dụng trường hợp riêng của định lý Hermite
$\lfloor 2x\rfloor =\lfloor x\rfloor+\lfloor x+\dfrac{1}{2}\rfloor\;,$ ta có
$\lfloor \dfrac{n+2^k}{2^{k+1}}\rfloor=\lfloor \dfrac{n}{2^{k+1}}+\dfrac{1}{2}\rfloor=\lfloor \dfrac{n}{2^k}\rfloor-\lfloor \dfrac{n}{2^{k+1}}\rfloor$
Vậy tổng cần tính là
$(\lfloor \dfrac{n}{2^0}\rfloor-\lfloor \dfrac{n}{2^1}\rfloor)+(\lfloor \dfrac{n}{2^1}\rfloor-\lfloor \dfrac{n}{2^2}\rfloor)+...+(\lfloor \dfrac{n}{2^k}\rfloor-\lfloor \dfrac{n}{2^{k+1}}\rfloor)+...=\lfloor n \rfloor=n$
(Số hạng $-\lfloor \dfrac{n}{2^{k+1}}\rfloor=0\;,$khi k đủ lớn)

Câu c
Áp dụng trường hợp riêng của định lý Hermite
$\lfloor 3x\rfloor =\lfloor x\rfloor+\lfloor x+\dfrac{1}{3}\rfloor+\lfloor x+\dfrac{2}{3}\rfloor=\lfloor x\rfloor+\lfloor x+\dfrac{1}{3}\rfloor+\lfloor x-\dfrac{1}{3}\rfloor +1\;,$ ta có
$\sum_{k=0}^{2009}(\lfloor \dfrac{2010+3^k}{3^{k+1}}\rfloor+\lfloor \dfrac{2010-3^k}{3^{k+1}}\rfloor)=\sum_{k=0}^{2009}(\lfloor \dfrac{2010}{3^{k+1}}+\dfrac{1}{3}\rfloor+\lfloor \dfrac{2010}{3^{k+1}}-\dfrac{1}{3}\rfloor)\\ =\sum_{k=0}^{2009}(\lfloor 3.\dfrac{2010}{3^{k+1}}\rfloor-\lfloor \dfrac{2010}{3^{k+1}}\rfloor-1)=-2010+\sum_{k=0}^{2009}(\lfloor\dfrac{2010}{3^k}\rfloor-\lfloor\dfrac{2010}{3^{k+1}}\rfloor)\\ =-2010+\lfloor 2010\rfloor-\lfloor \dfrac{2010}{3^{2010}}\rfloor=0$

Bài viết đã được chỉnh sửa nội dung bởi hxthanh: 27-12-2010 - 21:45


#14
hxthanh

hxthanh

    Tín đồ $\sum$

  • Hiệp sỹ
  • 3921 Bài viết
:Rightarrow Giải Phương trình:

$8x^2-6x\lfloor x\rfloor+\lfloor x\rfloor^2-3=0$

:Rightarrow Ai "chém" chuẩn nhất nào? (có 4 nghiệm tất cả đấy!)

Bài viết đã được chỉnh sửa nội dung bởi hxthanh: 10-01-2011 - 15:55


#15
NguyThang khtn

NguyThang khtn

    Thượng úy

  • Hiệp sỹ
  • 1468 Bài viết

:Rightarrow Giải Phương trình:

$8x^2-6x\lfloor x\rfloor+\lfloor x\rfloor^2-3=0$

:Rightarrow Ai "chém" chuẩn nhất nào? (có 4 nghiệm tất cả đấy!)


It is difficult to say what is impossible, for the dream of yesterday is the hope of today and the reality of tomorrow

 


#16
hxthanh

hxthanh

    Tín đồ $\sum$

  • Hiệp sỹ
  • 3921 Bài viết

:Rightarrow Giải Phương trình:

$8x^2-6x\lfloor x\rfloor+\lfloor x\rfloor^2-3=0$

:Rightarrow Ai "chém" chuẩn nhất nào? (có 4 nghiệm tất cả đấy!)

@bboy114crew
Tex đang lỗi thì phải!
Đề bài anh gõ LaTeX hẳn hoi, tưởng em vào giải, ngờ đâu lại ... quote bài anh sang Tex. Rồi chẳng ai thấy cái đề ra sao nữa
Bó tay!

#17
hxthanh

hxthanh

    Tín đồ $\sum$

  • Hiệp sỹ
  • 3921 Bài viết
Giải Phương trình:
$8x^2-6x\lfloor x\rfloor+\lfloor x\rfloor^2-3=0$

Xem phương trình trên là bậc 2 ẩn $\lfloor x\rfloor$, ta có:
$\Delta'=(-3x)^2-(8x^2-3)=x^2+3>0$
Nên phương trình có 2 nghiệm là $\lfloor x\rfloor_{1,2}=3x\pm \sqrt{x^2+3}\;\;:leq$
Theo định nghĩa phần nguyên thì phải có:
$x-1<\lfloor x\rfloor \le x$
Từ đó suy ra các bất phương trình sau:
$\left[\begin{gathered}x-1 < 3x+\sqrt{x^2+3} \le x \\ x-1 < 3x-\sqrt{x^2+3} \le x \end{gathered}\right. \Leftrightarrow \left[\begin{gathered}-1 < 2x+\sqrt{x^2+3} \le 0 \\ -1 < 2x-\sqrt{x^2+3} \le 0 \end{gathered}\right. \Leftrightarrow \left[\begin{gathered}\dfrac{-(\sqrt{10}+2)}{3} < x \le -1 \;\;(1)\\ \dfrac{\sqrt{10}-2}{3} < x \le 1 \;\;(2) \end{gathered}\right. $
$(1) \Rightarrow \left[\begin{gathered}\lfloor x\rfloor=-1 \\ \lfloor x\rfloor=-2 \end{gathered}\right.$ thay vào $:geq$ ta được $ \left[\begin{gathered}3x+\sqrt{x^2+3}=-2\\3x+\sqrt{x^2+3}=-1\end{gathered}\right. $

$(2) \Rightarrow \left[\begin{gathered}\lfloor x\rfloor=0 \\ \lfloor x\rfloor=1 \end{gathered}\right.$ thay vào $:rolleyes:$ ta được $ \left[\begin{gathered}3x-\sqrt{x^2+3}=0\\3x-\sqrt{x^2+3}=1\end{gathered}\right. $

Nghiệm của 4 phương trình thu được là nghiệm của phương trình ban đầu
Cụ thể 4 nghiệm là $-\dfrac{3+\sqrt{7}}{4},\;\;-1,\;\;\dfrac{\sqrt{6}}{4},\;\;1$
--------------
Cũng có thể, ngay từ đầu giải theo x nhưng phức tạp hơn!

#18
hxthanh

hxthanh

    Tín đồ $\sum$

  • Hiệp sỹ
  • 3921 Bài viết
Tính các tổng sau:

$a.\;\;\;\;\;S_n=\sum_{k=0}^n \lfloor\sqrt{k}\rfloor$

$b.\;\;\;\;\;S_n=\sum_{k=0}^n \lfloor\sqrt{2k}\rfloor $

#19
Nguyễn Thái Vũ

Nguyễn Thái Vũ

    Thiếu úy

  • Thành viên
  • 684 Bài viết
Phần nguyên đây khá hay và quen thuộc.
CMR: $(2m)!(2n)!$ chia hết cho $m!n!(m+n)!$ với m,n nguyên dương.

#20
hxthanh

hxthanh

    Tín đồ $\sum$

  • Hiệp sỹ
  • 3921 Bài viết

Phần nguyên đây khá hay và quen thuộc.
CMR: $(2m)!(2n)!$ chia hết cho $m!n!(m+n)!$ với m,n nguyên dương.

Bài này tôi nhớ không nhầm thì là của IMO 1972 thì phải
Điều cốt lõi ở đây là BĐT $\lfloor 2x\rfloor+\lfloor 2y\rfloor \ge \lfloor x\rfloor+\lfloor y\rfloor+\lfloor x+y\rfloor$ (1)
Trước hết ta CM BĐT (1)
Đặt $x=\lfloor x\rfloor+u;\;\;y=\lfloor y\rfloor+v$ với $0\le u;v<1$ (u, v là các phần lẻ)
Khi đó (1) tương đương với
$\left\lfloor 2\lfloor x\rfloor+2u\right\rfloor+\left\lfloor 2\lfloor y\rfloor+2v\right\rfloor\ge \left\lfloor \lfloor x\rfloor+u\right\rfloor+\left\lfloor \lfloor y\rfloor+v\right\rfloor+\left\lfloor \lfloor x\rfloor+\lfloor y\rfloor+u+v\right\rfloor$
$\Leftrightarrow \lfloor 2u\rfloor+\lfloor 2v\rfloor\ge\lfloor u+v\rfloor$
Vì $0\le u+v<2 \Rightarrow \lfloor u+v\rfloor=\{0,1\}$ (0 hoặc 1)
- Nếu $\lfloor u+v\rfloor=0$ thì $\lfloor u\rfloor=\lfloor v\rfloor=0 \Rightarrow $ BĐT hiển nhiên đúng
- Nếu $\lfloor u+v\rfloor=1$ thì ít nhất trong 2 số u, v có 1 số lớn hơn hoặc bằng $\dfrac{1}{2}$ giả sử $u\ge\dfrac{1}{2}$
Khi đó $\lfloor 2u\rfloor+\lfloor 2v\rfloor\ge \left\lfloor2.\dfrac{1}{2}\right\rfloor=1=\lfloor u+v\rfloor$
Ta có đpcm (1)
Trở lại bài toán:
Giả sử $p$ là một số nguyên tố bất kì. Khi đó số mũ của $p$ trong phân tích tiêu chuẩn của (2m)! (2n)! là
$A=\sum_{k=1}^{\infty} \left({\left\lfloor\dfrac{2m}{p^k}\right\rfloor+\left\lfloor\dfrac{2n}{p^k}\right\rfloor}\right)$
Còn số mũ của $p$ trong phân tích tiêu chuẩn của $m!n!(m+n)!$ là
$B=\sum_{k=1}^{\infty} \left({\left\lfloor\dfrac{m}{p^k}\right\rfloor+\left\lfloor\dfrac{n}{p^k}\right\rfloor+\left\lfloor\dfrac{m+n}{p^k}\right\rfloor}\right)$
Theo BĐT (1) ta có
$\left\lfloor\dfrac{2m}{p^k}\right\rfloor+\left\lfloor\dfrac{2n}{p^k}\right\rfloor\ge\left\lfloor\dfrac{m}{p^k}\right\rfloor+\left\lfloor\dfrac{n}{p^k}\right\rfloor+\left\lfloor\dfrac{m+n}{p^k}\right\rfloor$
Do đó $A \ge B$
Bài toán được chứng minh!





Được gắn nhãn với một hoặc nhiều trong số những từ khóa sau: Floor Function, Phần nguyên

0 người đang xem chủ đề

0 thành viên, 0 khách, 0 thành viên ẩn danh